Partage
  • Partager sur Facebook
  • Partager sur Twitter

Les exercices d'olympiade

les plus tordus !!

14 août 2012 à 4:07:47

@Elionor ,est ce que tu peux posté ta solution ?
  • Partager sur Facebook
  • Partager sur Twitter
Anonyme
14 août 2012 à 15:36:19

Bonjour,

Citation : oty

@Elionor le triplet (2,3,3)est solution mais ne figure pas dans le triplet que tu as trouver .



quelque chose m'échappe , si <math>\(x=2, z=1\)</math>, le triplet d'Elionor donne bien (2,3,3) (?non :( )

Guidé par le triplet affiché par Elionor, ainsi que, je dois le dire , un problème que je connaissais, a priori plus simple ...(IMO 2006 quand même :diable: ),mais dont on pouvait s'inspirer , où il était demandé de trouver <math>\(x\)</math> tel que <math>\(1+2^x+2^{2x+1}\)</math> soit carré parfait.

Je retrouve sous une forme équivalente les triplets d'Elionor, :D , ...petit doute sur ma preuve qu'il n'y en pas d'autres :o .

Si x,y,z est solution , toute permutation est solution je suppose <math>\(x<y<z\)</math> sans perte de généralité, les cas d'égalités sont traités à la fin.
On doit alors avoir <math>\(4^x+4^y+4^z=4^x(1+4^{y-x}+4^{z-x})=n^2, y-x>0,z-x>0\)</math>
<math>\(4^x=2^{2x}\)</math>étant un carré, on doit avoir le second terme du produit carré parfait soit
<math>\(1+4^{y-x}+4^{z-x}=k^2\)</math>
Mais le membre de gauche est impair donc <math>\(k\)</math> est impaire . soit <math>\(k=2p+1\)</math> et donc :
<math>\(4^{y-x}+4^{z-x}=(k+1)(k-1)=4p(p+1)\)</math>
Posons <math>\(y-x= u,\; z-x =v,\; u<v\)</math>
Aprés une nouvelle factorisation, on obtient:
<math>\(2^{2u}(1+2^{2(v-u)})=4p(p+1)\)</math>

Tout entier <math>\(p\)</math> se décompose ( de façon unique) en <math>\(p=2^rs\)</math>, <math>\(s\)</math> impair.
La relation peut donc s'écrire:
<math>\(2^{2u-2}(1+2^{2(v-u)})=2^rs(2^rs+1)\)</math>
On a de part et d'autre le produit d'une puissance de 2 par un nombre impair. L'unicité implique:
<math>\(2u-2=r\)</math> et <math>\(1+2^{2(v-u)})=s(2^rs+1)\)</math>

On voit donc que <math>\(r\)</math> doit être pair soit <math>\(r=2m\)</math> et <math>\(u=m+1\)</math>
Dans la seconde relation ,
- si <math>\(s=1\)</math> , 0n en déduit <math>\(2(v-u)=2m\)</math> donc <math>\(v=2m+1\)</math>
soit en revenant au variables, <math>\(y=x+m+1,\;z=x+2m+1\)</math>, ce qui équivaut au triplet d'Elionor avec <math>\(z=m+1\)</math>

- si <math>\(s > 1\)</math>, on a nécessairement <math>\(v>2m+1\)</math> puisque le second membre de <math>\(1+2^{2(v-u)})=s(2^rs+1)\)</math>
étant plus grand, le premier membre ne peut l'être que si <math>\(v\)</math> est supérieur à la solution pour <math>\(s=1\)</math>, <math>\(u\)</math>restant égal à <math>\(m+1\)</math> nécessairement.

Posons <math>\(v=2m+1+w\)</math>, alors <math>\(1+2^{2(m+w)})=s(2^{2m}s+1)=s^22^{2m}+s\)</math>
Mais <math>\(s\)</math> étant impair, et la décomposition d'un entier en puissances de 2 étant unique, cette égalité est impossible si <math>\(s \neq 1\)</math> et alors <math>\(w= 0\)</math>.

Finalement je trouve comme seules solutions, les triplets anoncés par Elionor.

Cas d'égalité
- si <math>\(x=y=z\)</math>, o obtient <math>\(3.4^x\)</math>, il est clair qu'il n'y a pas de solution,

- si <math>\(x<y=z\)</math> , ce qui précéde me semble valable et correspond à <math>\(m=0\)</math> et la solution (<math>\(x,x+1,x+1)\)</math>,

-si <math>\(x=y<z\)</math>, on cherche <math>\(4^x(2+4^{z-x})=n^2\)</math> qui n'a pas de solution.
  • Partager sur Facebook
  • Partager sur Twitter
14 août 2012 à 16:49:30

Citation : nabucos

Bonjour,

Citation : oty

@Elionor le triplet (2,3,3)est solution mais ne figure pas dans le triplet que tu as trouver .



quelque chose m'échappe , si <math>\(x=2, z=1\)</math>, le triplet d'Elionor donne bien (2,3,3) (?non :( )


Bonjour ,
oui tu as raison Nabucos BRAVO belle solution ,il s’avère que le résultat d'ELionor est correcte , le triplet au qu'elle vous avez aboutis est équivalent a celui ci <math>\((x,y,2y-x-1)\)</math> qui est la solution du problème ;) : <math>\(4^x+4^y+4^z=4^x+4^y+4^{2y-x-1}=(2^x+2^{2y-x-1})^2\)</math>^^ , qui as eu le don de me faire douté de votre résultat je m'en excuse .
  • Partager sur Facebook
  • Partager sur Twitter
14 août 2012 à 18:01:35

On reprends avec un simple IMO de 2010,

IMO 2010 P1



Trouvez toutes les fonctions <math>\(f:\mathbb{R}\rightarrow\mathbb{R}\)</math> tels que <math>\(\forall x,y\in\mathbb{R}\)</math> l'égalité suivante est vérifiée <math>\(f(\left\lfloor x\right\rfloor y)=f(x)\left\lfloor f(y)\right\rfloor\)</math> .

PS: Ce serai agréable de voir quelques simples combinatoires ici de temps en temps, have fun. ^^
  • Partager sur Facebook
  • Partager sur Twitter
17 août 2012 à 1:58:07

Citation : Elionor

On reprends avec un simple IMO de 2010,

IMO 2010 P1



Trouvez toutes les fonctions <math>\(f:\mathbb{R}\rightarrow\mathbb{R}\)</math> tels que <math>\(\forall x,y\in\mathbb{R}\)</math> l'égalité suivante est vérifiée <math>\(f(\left\lfloor x\right\rfloor y)=f(x)\left\lfloor f(y)\right\rfloor\)</math> .

PS: Ce serai agréable de voir quelques simples combinatoires ici de temps en temps, have fun. ^^


on pose : <math>\(P(x,y)\)</math> l'assertion <math>\(f(\left\lfloor x\right\rfloor y)=f(x)\left\lfloor f(y)\right\rfloor\)</math> .
notant que : <math>\(f(\lfloor{x} \rfloor y )=f(\lfloor \lfloor x \rfloor \rfloor y)=f(\lfloor x \rfloor )\lfloor f(y) \rfloor\)</math> .
d'ou <math>\((f(x)-f(\lfloor x \rfloor)) \lfloor f(y) \rfloor=0\)</math> .
1er cas : si <math>\(\lfloor f(y) \rfloor=0\)</math> quelque soit <math>\(y\)</math> ,
alors <math>\(P(1,x)\)</math> donne <math>\(f(x)=0\)</math> quelque soit x .
2e cas :
si : <math>\(f(x)=f(\lfloor x \rfloor)\)</math> quelque soit x .
<math>\(P(x,1)\)</math> implique : <math>\(f(x)(\lfloor f(1) \rfloor - 1)=0\)</math> on fixe x tel que <math>\(f(x) \neq 0\)</math> , on obtient <math>\(\lfloor f(1) \rfloor=1\)</math>, il s'ensuit que
<math>\(1\leq f(1) < 2\)</math> .
Remarquant qu'on peut écrire P(x,y) de la maniéré suivante :
<math>\(f(\lfloor x \rfloor y)=f(x)\lfloor f(\lfloor y \rfloor) \rfloor\)</math> , et donc
<math>\(P(n,\frac{1}{n})\)</math> implique : <math>\(f(1)=f(n)\lfloor f(0) \rfloor\)</math> et par conséquent <math>\(\lfloor f(0) \rfloor\neq0\)</math> , ce qui nous permet de conclure car
<math>\(P(x,0)\)</math> nous donne <math>\(f(x)=\frac{f(0)}{\lfloor f(0) \rfloor}=k \in [1,2[\)</math> .
Réciproquement c'est deux types de fonctions vérifient l’équation .

  • Partager sur Facebook
  • Partager sur Twitter
18 août 2012 à 19:18:15

Bonsoir , Voici un nouveau problème très intéressant :

Combinatoire :


Des chiffres <math>\(1,2,...9\)</math> , on écrit tous les nombres former par ces neuf chiffres (les neuf distincts) , et nous les ordonnons de la manière suivante : <math>\(123456789\)</math> ,<math>\(123456798\)</math>, .., <math>\(987654321\)</math> .
Quelle est le <math>\(100000th\)</math> nombre ?
  • Partager sur Facebook
  • Partager sur Twitter
18 août 2012 à 19:24:56

C'est le même exercice qu'ici : http://projecteuler.net/problem=24,

la méthode est : la 8! ème permutation est 198765432, la 8!+1 est 213456789 donc la 9!+9! permutation est 298765431 etc etc jusqu'à ce qu'on arrive a un nombre , après itération et l'écriture de 10^6 sous forme de combinaison de factorielle on arrive au résultat .
  • Partager sur Facebook
  • Partager sur Twitter
18 août 2012 à 20:36:55

J'explique si j'ai bon :
358926471?
  • Partager sur Facebook
  • Partager sur Twitter
18 août 2012 à 21:50:57

tu peux expliquer Leoleo , c'est le bon résultat :) .
  • Partager sur Facebook
  • Partager sur Twitter
18 août 2012 à 21:59:00

J'ai fait à la main:

358926714, soit celle après @leoleo, voici ma méthode "manuelle", je sais pas si c'est hyper clair:

Pour un nombre quelconque de <math>\(n\)</math> chiffres, il y a <math>\(n!\)</math> permutations possibles.
En décomposant <math>\(123456789\)</math> en groupes qui permutent de <math>\(n\)</math> chiffres, donc <math>\(*******89\)</math>, <math>\(******789\)</math>, <math>\(*****6789\)</math>,etc. par exemple, on sait qu'ils peuvent effectuer n! permutations (avant d'augmenter d'une unité le groupe supérieur).

Il faut donc savoir combien de permutations par groupe on été effectuées jusqu'au 100000ième.
J'ai décomposé 100000 en factorielles, ça nous donne
<math>\(2*8! + 3*7! + 5*6! + 5*5! + 1*4! + 2*3! + 2*2! + 0*1! + 0\)</math>

Après, je suis partit de 123456789 et j'ai appliqué le nombre de permutations par groupes.

On a <math>\(2*8!\)</math> permutations, donc le premier chiffre est 1+2, soit 3
<math>\(3*7!\)</math>, donc 1+3 et +1, car 3 est déjà utilisé, donc 5,
etc...


EDIT: Ah zut, 1 de trop donc
  • Partager sur Facebook
  • Partager sur Twitter
Graphiste/Développeur/Bidouilleur
18 août 2012 à 22:27:34

Au nouveau visiteurs du topic, je voudrais très bien connaitre de vos suggestions d'exercice pour activer le topic, vue que le problème d'Oty a miraculeusement rapporté deux visiteurs, est-ce donc une contrainte d'exercice ?. Merci de répondre :D .
  • Partager sur Facebook
  • Partager sur Twitter
18 août 2012 à 22:36:22

Citation : Elionor

Au nouveau visiteurs du topic, je voudrais très bien connaitre de vos suggestions d'exercice pour activer le topic, vue que le problème d'Oty a miraculeusement rapporté deux visiteurs, est-ce donc une contrainte d'exercice ?. Merci de répondre :D .


Desar et Leoleo ne sont pas de nouveau visiteur , c'est juste qu'il viennent de refaire apparition . Bienvenu :) .
  • Partager sur Facebook
  • Partager sur Twitter
18 août 2012 à 23:39:33

@Desare : J'étais tombé là dessus au début, mais j'ai ensuite essayé avec un exemple simple pour me convaincre de ma méthode. Donc j'ai rectifié après coup. :-° Sinon, j'ai fait pareil, et je pense pas que d'autres méthodes existent vraiment, si? (à part avec un algo bien sûr...)

@Elionor : il faudrait juste des exos à ma portée. ;) En fait je viens toujours ici mais j'ai la flemme de faire les exos... Celui-là me paraissait simple, donc je l'ai fait.
  • Partager sur Facebook
  • Partager sur Twitter
19 août 2012 à 1:15:40

@Elionor: Beh moi c'est pareil que @leoleo, la majorité des exercices, je ne peux pas les résoudre, à moins de chercher sur Internet les notations que je connais pas etc. (et j'ai donc la flemme)

@leoleo: beh j'ai dis "fait main" parce que ça me paraît un peu barbare de trouver la décomposition factorielle en tâtonnant, mais soit, tant mieux si c'est la seule méthode. Pour la permutation de trop je comprends à moitié ^^
  • Partager sur Facebook
  • Partager sur Twitter
Graphiste/Développeur/Bidouilleur
19 août 2012 à 21:18:33

Inégalité, Maroc 2009



Soit <math>\(a_1,a_2,\cdots ,a_n > 0\)</math> tels que : <math>\(a_1a_2 \cdots a_n=1\)</math> , démontrer que :

<math>\((4+a_1)(4+a_2)\cdots(4+a_n) \geq 5^n\)</math>
  • Partager sur Facebook
  • Partager sur Twitter
19 août 2012 à 21:38:37

Citation : Elionor

Inégalité, Maroc 2009



Soit <math>\(a_1,a_2,\cdots ,a_n > 0\)</math> tels que : <math>\(a_1a_2 \cdots a_n=1\)</math> , démontrer que :

<math>\((4+a_1)(4+a_2)\cdots(4+a_n) \geq 5^n\)</math>


<math>\(4+a_{i}=1+1+1+1+a_{i} \geq{ 5\sqrt[5]a_{i}\)</math>:o
  • Partager sur Facebook
  • Partager sur Twitter
20 août 2012 à 2:48:23

Citation : oty

Citation : Elionor

Inégalité, Maroc 2009



Soit <math>\(a_1,a_2,\cdots ,a_n > 0\)</math> tels que : <math>\(a_1a_2 \cdots a_n=1\)</math> , démontrer que :

<math>\((4+a_1)(4+a_2)\cdots(4+a_n) \geq 5^n\)</math>


<math>\(4+a_{i}=1+1+1+1+a_{i} \geq{ 5\sqrt[5]a_{i}\)</math>:o



Mon horrible démonstration :-° :
<math>\((4+a_1)(4+a_2)\cdots(4+a_n)=4^n + 1 + 4( \sum a_1a_2 \cdots a_{n-1} ) +4^2( \sum a_1a_2 \cdots a_{n-2} ) +\cdots +4^{n-1}( \sum a_1)\)</math>

Par AM-GM :
<math>\(\sum a_1 \cdots a_k \geq \binom{n}{k}.\sqrt[\binom{n}{k}]{(a_1a_2 \cdots a_{n})^{\binom{n}{k}-1}}= \binom{n}{k}\)</math>

donc :<math>\(4^n + 1 + 4( \sum a_1a_2 \cdots a_{n-1} ) +4^2( \sum a_1a_2 \cdots a_{n-2} ) +\cdots +4^{n-1}( \sum a_1) \geq 4^n + 1 + 4.\binom{n}{n-1}+4^2\binom{n}{n-2}+\cdots =(4+1)^n=5^n\)</math>

Très horrible comme solution, je l'avoue >_< .
  • Partager sur Facebook
  • Partager sur Twitter
20 août 2012 à 19:30:56

Nombres Premiers :


Trouver tous les nombres premiers <math>\(p\)</math> et <math>\(q\)</math> vérifiant l'équation suivante :

<math>\((p+q)^{p}=(q-p)^{2q-1}\)</math> .
  • Partager sur Facebook
  • Partager sur Twitter
Anonyme
5 septembre 2012 à 9:49:26

Bonjour,

retour de vacances :soleil: pour cet exo qui semble à l'abandon. Une tentative donc...

Citation : oty

Nombres Premiers :


Trouver tous les nombres premiers <math>\(p\)</math> et <math>\(q\)</math> vérifiant l'équation suivante :

<math>\((p+q)^{p}=(q-p)^{2q-1}\)</math> .


L'équation à vérifier implique <math>\(2 \leq p<q\)</math>.
Notant <math>\(d\)</math> le PGCD de <math>\(p+q,p-q\)</math>, on a :
<math>\(p+q=d.n\)</math>, <math>\(q-p=d.m\)</math> avec <math>\(PGCD(n,m)=1\)</math>, <math>\(n>m\)</math> et <math>\(2q=d(n+m),\; 2p=d(n-m)\)</math>
<math>\(p,q\)</math> premiers impliquent que <math>\(d=1\)</math> ou <math>\(d=2\)</math>.
<math>\(d=1\)</math> conduit à une impossibilité car on devrait avoir <math>\(n^p=m^{2q-1}\)</math>, <math>\(m^p\)</math> divise le second membre, donc <math>\(m\)</math> devrait diviser <math>\(n\)</math> or <math>\(PGCD(n,m)=1\)</math>.On devrait avoir <math>\(m=1\)</math> qui implique une absence de solution.

donc <math>\(d=2\)</math> et l'équation de l'énoncé peut alors s'écrire:
<math>\((2n)^{p}=(2m)^{2q-1}\)</math> ou encore <math>\((n)^{p}=2^{2q-p-1}m^{2q-1}\)</math>
Comme précédemment, <math>\(m^p\)</math> divise le second membre, donc <math>\(m\)</math> doit diviser <math>\(n\)</math>
Donc <math>\(m=1\)</math> , ce qui implique dans ce cas pour <math>\(n\)</math> :
<math>\(n^{n-1}=2^{n+2}\)</math> et la seule possibilité est <math>\(n=4\)</math>
D'où le couple <math>\((p,q)=(3,5)\)</math>. On vérifie que c'est bien une solution (chaque membre vaut 64)
C'est a priori le seul,...s'il n'y a pas de faille dans la démonstration. :p
  • Partager sur Facebook
  • Partager sur Twitter
7 septembre 2012 à 1:56:08

Bravo Nabucos :) , on peut aussi remarquer que
<math>\(q>p\)</math> implique que : <math>\(2q-1>p\)</math> , il s'ensuit que :
<math>\((\frac{p+q}{q-p})^{p}\in N\)</math> et par conséquent <math>\(q-p|p+q\)</math> ,d'ou <math>\(q-p|2p\)</math> , mais les seuls diviseurs de <math>\(2p\)</math> sont <math>\(1,2,p,2p\)</math> le résultat en découle .

Nouveau style :


Trouvez la partie entière de la somme :
<math>\(\sum_{n=1}^{10^{9}} n^{-\frac{2}{3}}\)</math> .
  • Partager sur Facebook
  • Partager sur Twitter
Anonyme
12 septembre 2012 à 14:16:35

Bonjour,

Citation : oty

Nouveau style :


Trouvez la partie entière de la somme :

<math>\(\sum_{n=1}^{10^{9}} n^{-\frac{2}{3}}\)</math> .


réponse proposée:
<math>\(2997\)</math> ... :p
  • Partager sur Facebook
  • Partager sur Twitter
13 septembre 2012 à 16:20:39

Bonjour ,
Exactement Nabucos , Bravo ! :)
  • Partager sur Facebook
  • Partager sur Twitter
15 septembre 2012 à 0:38:19

Les suites :


<math>\((a_{n})\)</math> est une suite d'entiers naturels tel que : <math>\(a_{1}=1\)</math> , <math>\(a_{n+1}\leq 2n\)</math> pour tout entiers naturels <math>\(n\)</math> et <math>\((a_{n})\)</math>
est strictement croissante .
Démontrer que pour tout entier naturel <math>\(n\)</math> il existe deux termes <math>\(a_{p}\)</math> et <math>\(a_{q}\)</math> vérifiant : <math>\(a_{p}-a_{q}=n\)</math> .
  • Partager sur Facebook
  • Partager sur Twitter
15 septembre 2012 à 10:50:58

je suis pas sûr d'avoir compris le principe du problème, en gros si on veut <math>\(n=10\)</math>, <math>\(p=7\)</math> et <math>\(q=1\)</math> ?
  • Partager sur Facebook
  • Partager sur Twitter
Graphiste/Développeur/Bidouilleur
16 septembre 2012 à 19:19:39

Salut ,
je n'ai pas bien compris t'as remarque Desare :euh: , pourrais tu élaborer ? Merci .
  • Partager sur Facebook
  • Partager sur Twitter
16 septembre 2012 à 22:26:08

en fait je pense qu'il y a un truc qui l'échappe dans le problème, sinon <math>\(a_p - a_q = n\)</math> facilement, il suffit de prendre <math>\(p=\frac{n}{2}+2\)</math> et garder <math>\(q=1\)</math>, non ? (et retirer une unité aux impairs)
  • Partager sur Facebook
  • Partager sur Twitter
Graphiste/Développeur/Bidouilleur
16 septembre 2012 à 22:50:55

La condition c'est <math>\(a_{n+1}\leq 2n\)</math> et pas <math>\(a_{n+1}= 2n\)</math>
  • Partager sur Facebook
  • Partager sur Twitter
Anonyme
17 septembre 2012 à 0:02:49

Bonsoir,

Comme le rappelle Rushia, la condition est <math>\(a_{n+1} \leq 2n\)</math>
ce qui veut dire par exemple, pour les premiers termes que <math>\(a_1 =1\)</math>, <math>\(a_2 \leq 2\)</math>, <math>\(a_3 \leq 4\)</math> ,<math>\(a_4 \leq 6\)</math>, a<math>\(_5 \leq 8\)</math>, <math>\(a_6\leq 10\)</math>, etc ...
Par ailleurs l'autre condition <math>\([a_n]\)</math> doit être strictement croissante, ce qui limite les valeurs possibles
Par exemple nécessairement <math>\(a_2 = 2\)</math>, <math>\(a_3\)</math> peut valoir 3 ou 4, que <math>\(a_4\)</math> peut valoir 4,5, ou 6 si <math>\(a_3\)</math> vaut 3 et 5 ou 6 si <math>\(a_3\)</math> vaut 4 etc...

On se rend compte que l'arborescence des suites possibles jusqu'au rang n croit rapidement avec n et que l'on peut finalement construire un ensemble S infini de suites <math>\((a_n)\)</math> respectant la régle. ( sauf erreur, il y a déjà 14 débuts distincts de suites possibles pour les 5 premiers termes)

La question tel que je l'ai comprise est donc de montrer que pour toute suite arbitraire de S , il existe (au moins) deux éléments de la suite dont la différence est <math>\(n\)</math>, ceci pour tout rang <math>\(n\)</math>.

  • Partager sur Facebook
  • Partager sur Twitter
19 septembre 2012 à 20:53:13

Bonsoir ,
Alors aucune tentative o_O (c'est assez faisable )
  • Partager sur Facebook
  • Partager sur Twitter
20 septembre 2012 à 1:03:02

Je suis juste trop overbooké entre mes cours à Supélec, mes cours à GeorgiaTech et la vie de campus. Pas vraiment de temps à consacrer au site du zéro :( .
  • Partager sur Facebook
  • Partager sur Twitter